LSAT and Law School Admissions Forum

Get expert LSAT preparation and law school admissions advice from PowerScore Test Preparation.

User avatar
 chrisfromc123
  • Posts: 5
  • Joined: May 08, 2021
|
#87848
E makes perfect sense, but I still was hung up on why the province would spend out of province. But then you think of the amount of foreign aid that many countries give. The same principle can apply to provinces... wow this was a good one.

Logic is an abstract thing. Getting too hung up on our presumptions is really a killer.

Thanks powerscore team.
User avatar
 KwakuS
  • Posts: 35
  • Joined: Jun 03, 2021
|
#88221
Hello,

Is it fair to say that D is wrong because spending $600 million would still leave the province with a spending deficit? My reasoning kind of goes along these lines: Refunding $600 million would cause a budget deficit. Therefore, spending $600 million on infrastructure, even if it did create some jobs, would still leave the province with a budget deficit that would not stimulate economic growth.

Also, could we say that stimulating economic activity and saving money/making profit are the same thing?
 Robert Carroll
PowerScore Staff
  • PowerScore Staff
  • Posts: 1787
  • Joined: Dec 06, 2013
|
#88238
KwakuS,

Answer choice (D) is incorrect for a more straightforward reason. If the province did not refund the money, then the situation about which the author is making a conclusion wouldn't even happen. The author is only predicting that, if the refund is passed, there will be no net increase in spending. If the refund is abandoned, as answer choice (D) envisions, you can't strengthen or weaken the author's argument - the author is silent about what would happen in other situations.

Robert Carroll
 Prm013@bucknell.edu
  • Posts: 1
  • Joined: Mar 31, 2021
|
#96398
I'm going through the online resources and came across this question. I have been doing pretty well overall in weakening questions, but this question confused me. I read the explanation for this question, and I'm still confused. I chose D because it seemed possible to distribute the 600 million to both groups and wouldn't necessarily mean "there can be no resulting net increase in spending to stimulate the province’s economy." What if this distribution of the 600 million leads to a net increase in spending that stimulates the province's economy? I don't think this possibility is necessarily ruled out. :-?

Thanks
User avatar
 atierney
PowerScore Staff
  • PowerScore Staff
  • Posts: 215
  • Joined: Jul 06, 2021
|
#96485
Hello,

As Robert mentions above, the problem with D is that it doesn't actually address the validity of the author's conclusion. Notice that the conclusion is a conditional statement, it's about the likely consequences of a proposed action. D provides an alternative in which the proposed action doesn't occur, thus eliminating the ability to evaluate the author's conclusion. As an analogy, if I say if the superbowl is held on Sunday it will rain, but then the superbowl is cancelled, you'll never know if I was correct or not, even if it doesn't rain on Sunday. This is because the sufficient condition did not occur, so any statement about the necessary condition occurring is automatically irrelevant. As a side note, this occurs with any conditional statement any time the sufficient condition does not occur.

Let me know if you have any questions.
 olenka.ballena@macaulay.cuny.edu
  • Posts: 17
  • Joined: Feb 16, 2022
|
#96636
Hi Powerscore,

I got this question right (although I definitely spent way too much time on it), but I want to make sure I ultimately got it right for the correct reasons. My prephase essentially was the following: how can there still be a resulting net increase in spending with the tax refund?

As I went through the answer choices, I was focused on finding one that resulted in a total net increase (that's kind of the key idea I fixated on), and I was able to eliminate A-C since they seemed irrelevant to the argument. When I got stuck in between D and E, my reasoning was the following:

D) doesn't necessarily indicate a resulting net increase, as the $600 million dollars are still being spend just elsewhere and there's no guarantee that this will somehow result in a net increase?

E) just seemed stronger to me regardless of what D meant because a saving of $600 million dollars, no matter where it came from, would automatically result in a total net increase, without any new taxes or workers being fired.

Is this thought process correct? I wasn't sure because a lot of the explanations on this form got into a lot more depth than what my initial logic was when doing this question (I'm also not sure if the logic I used was effective as I spent longer than I needed on the question as well). Thanks in advance!
 Adam Tyson
PowerScore Staff
  • PowerScore Staff
  • Posts: 5153
  • Joined: Apr 14, 2011
|
#97303
Correct, olenka, and well done! It really can be that simple - the correct answer will show that we can have this tax refund with no new taxes and with nobody losing their job, resulting in a net increase in spending WITHIN the province. Cutting costs OUTSIDE the province would allow that to happen!

Get the most out of your LSAT Prep Plus subscription.

Analyze and track your performance with our Testing and Analytics Package.